ChaseDream
搜索
返回列表 发新帖
00:00:00

Three large companies and seven small companies currently manufacture a product with potential military applications. If the government regulates the industry, it will institute a single set of manufacturing specifications to which all ten companies will have to adhere. In this case, therefore, since none of the seven small companies can afford to convert their production lines to a new set of manufacturing specifications, only the three large companies will be able to remain in business.

Which of the following is an assumption on which the author's argument relies?

正确答案: E

更多相关帖子

524

帖子

15

好友

4712

积分

ChaseDream

注册时间
2003-03-17
精华
8
解析
查看: 2486|回复: 2
打印 上一主题 下一主题

没讨论过-GWD4-38

[复制链接]
跳转到指定楼层
楼主
发表于 2010-6-16 17:22:58 | 只看该作者 回帖奖励 |倒序浏览 |阅读模式
Three large companies and seven small companies currently manufacture a product with potential military applications.If the government regulates the industry, it will institute a single set of manufacturing specifications to which all ten companies will have to adhere.In this case, therefore, since none of the seven small companies can afford to convert their production lines to a new set of manufacturing specifications, only the three large companies will be able to remain in business.

Which of the following is an assumption on which the author’s argument relies?



A.None of the three large companies will go out of business if the government does not regulate the manufacture of the product.

B.It would cost more to convert the production lines of the small companies to a new set of manufacturing specifications than it would to convert the production lines of the large companies.

C.Industry lobbyists will be unable to dissuade the government from regulating the industry.

D.Assembly of the product produced according to government manufacturing specifications would be more complex than current assembly procedures.

E.None of the seven small companies currently manufactures the product to a set of specifications that would match those the government would institute if the industry were to be regulated.

红色为GWD的答案-E,蓝色为我的答案-A,为什么我觉得A和E都是正确的呢?A从3家大公司的角度出发,如果这3家大公司不管政府是否出台相关政策也会不再继续经营了,那么结论就是不成立的! 请大家帮助一下!
收藏收藏 收藏收藏
沙发
发表于 2010-6-16 18:09:29 | 只看该作者
A项讨论的是大公司情况,还是没有REGULATION之前的;题干的结论是有REGULATION之后的小公司情况。这两者没有逻辑联系。

A选项是无关项。
板凳
 楼主| 发表于 2010-6-19 22:02:07 | 只看该作者
回复kevin1473 :

谢谢你的回复,但是我不是很同意你关于题干结论的一些看法呢,我觉得题目的结论是与7家小公司和3家大公司的情况有关,即:7家小公司不能持续经营,最后指剩下3家大公司在这个行业中。

open to discussion
您需要登录后才可以回帖 登录 | 立即注册

Mark一下! 看一下! 顶楼主! 感谢分享! 快速回复:

手机版|ChaseDream|GMT+8, 2024-9-17 05:41
京公网安备11010202008513号 京ICP证101109号 京ICP备12012021号

ChaseDream 论坛

© 2003-2023 ChaseDream.com. All Rights Reserved.

返回顶部